Đến nội dung

Hình ảnh

Chứng minh rằng: $(abc)^{\frac{a+b+c}{3}}\leq a^ab^bc^c$


  • Please log in to reply
Chủ đề này có 4 trả lời

#1
thanhelf96

thanhelf96

    Trung sĩ

  • Thành viên
  • 155 Bài viết
1) Cho a,b,c là ba số dương. Chứng minh rằng: $(abc)^{\frac{a+b+c}{3}}\leq a^ab^bc^c$

2) Cho 3 số dương x,y,z thỏa mãn : $x+3y+5z\leq 3$. Chứng minh rằng:
$3xy\sqrt{625z^4+4}+15yz\sqrt{x^4+4}+5zx\sqrt{81y^4+4}\geq 45\sqrt{5}xyz$

sống là cho đâu chỉ nhận riêng mình  :icon6:


#2
thanhelf96

thanhelf96

    Trung sĩ

  • Thành viên
  • 155 Bài viết
các bạn có thể tham khảo lời giải câu 2 tại đây :
còn câu 1 thì mình chịu :(

sống là cho đâu chỉ nhận riêng mình  :icon6:


#3
WhjteShadow

WhjteShadow

    Thượng úy

  • Phó Quản lý Toán Ứng dụ
  • 1323 Bài viết

1) Cho a,b,c là ba số dương. Chứng minh rằng: $(abc)^{\frac{a+b+c}{3}}\leq a^ab^bc^c$

Do $\left(\frac{a+b+c}{3}\right)^3\geq abc$ nên ta sẽ chứng minh:
$$a^ab^bc^c\geq \left(\frac{a+b+c}{3}\right)^{a+b+c}$$
Lấy Logarit Nepe 2 vế, ta cần chứng minh:
$$ a.\ln a+b.\ln b+c.\ln c\geq (a+b+c).\ln \left(\frac{a+b+c}{3}\right)$$
Xét hàm số $f(a)=a.\ln a$, ta có $f''(a)=\frac{1}{a}>0 \forall a>0$ nên hàm số $f(a)=a.\ln a$ lõm trên $(0;\infity)$
Áp dụng bất đẳng thức $Jensen$ ta có:
$$f(a)+f(b)+f \left(c\right)\geq 3f \left(\frac{a+b+c}{3}\right)$$
$$\Leftrightarrow a.\ln a+b.\ln b+c.\ln c\geq (a+b+c).\ln \left(\frac{a+b+c}{3}\right)$$
Ta có điều phải chứng minh. Đẳng thức xảy ra khi $a=b=c$ $\square$
“There is no way home, home is the way.” - Thich Nhat Hanh

#4
anhminhkhon

anhminhkhon

    Hạ sĩ

  • Thành viên
  • 99 Bài viết
câu 2 chắc sử dụng côsi
sau đấy thì mình chịu

Bài viết đã được chỉnh sửa nội dung bởi anhminhkhon: 20-01-2013 - 21:11


#5
thanhelf96

thanhelf96

    Trung sĩ

  • Thành viên
  • 155 Bài viết

Do $\left(\frac{a+b+c}{3}\right)^3\geq abc$ nên ta sẽ chứng minh:
$$a^ab^bc^c\geq \left(\frac{a+b+c}{3}\right)^{a+b+c}$$
Lấy Logarit Nepe 2 vế, ta cần chứng minh:
$$ a.\ln a+b.\ln b+c.\ln c\geq (a+b+c).\ln \left(\frac{a+b+c}{3}\right)$$
Xét hàm số $f(a)=a.\ln a$, ta có $f''(a)=\frac{1}{a}>0 \forall a>0$ nên hàm số $f(a)=a.\ln a$ lõm trên $(0;\infity)$
Áp dụng bất đẳng thức $Jensen$ ta có:
$$f(a)+f(b)+f \left(c\right)\geq 3f \left(\frac{a+b+c}{3}\right)$$
$$\Leftrightarrow a.\ln a+b.\ln b+c.\ln c\geq (a+b+c).\ln \left(\frac{a+b+c}{3}\right)$$
Ta có điều phải chứng minh. Đẳng thức xảy ra khi $a=b=c$ $\square$

Em mới học lớp 11 có cách nào dành cho lớp 11 không anh? :mellow:

sống là cho đâu chỉ nhận riêng mình  :icon6:





1 người đang xem chủ đề

0 thành viên, 1 khách, 0 thành viên ẩn danh